find the value of the second in this sequence

34 __ 84 109 134

Answers

Answer 1

Answer:

the answer is 59

Step-by-step explanation:

your welcome


Related Questions

work out the perimeter of the semi-circle take pi to be 3.142​

Answers

Answer:
P = 25.71 cm
Step-by-step explanation:
Perimeter of semicircle = πD/2
Where D=10 cm
P = (3.142)(10)/2
P = (3.142)(5)
P = 15.71 cm
Complete Perimeter of Semicircle = P + Diameter
= 15.71 + 10
= 25.71 cm

I don’t know what to do please help

Answers

Answer:

f(2)=0

Step-by-step explanation:

You plug in 2 for any X's in the equation

f(2)=2^2-4

If f(x)=x^2-4 and x=2
Then f(2)=2^2-4.
= 4-4
=0
Don’t write this in answer just to explain:the value of x is 2 so in the eq
X^2-4 so we write 2 where the x is
= 2^2-4 ...now solve it as above

Use the diagram below to answer the following questions.
The infield of a baseball field is a square. The distance from home plate to first base is 90 ft.
2nd
8a. What is the distance from home plate to second base?
pitchers mound
3rd
1st
90 ft
8b. What is the distance from third base to first base?
Home
8c. If the pitcher's mound is 60 ft 6 in from home plate, is it the midpoint of the diagonal from home
plate to second base?

Answers

Answer:

Step-by-step explanation:

Distance from home plate to second base = 90√2 feet

Distance from third base to first base = 90√2 feet

60 ft 6 in = 60.5 ft

90√2 /2 = 45√2 ≅ 63.6 ft ≠ 60.5 ft

The pitcher's mound is not at the midpoint.

Can someone please help me

Answers

Answer:

A) 23cm.

B) 32.0625 square cm.

Step-by-step explanation:

Perimeter: Just add all the sides together like so:

4.75x2 + 6.75x2 = 23cm.

Area: Multiply the base and height together:

4.75 x 6.75 = 32.0625 square cm.

23cm

4.75 + 4.75 = 9.50

6.75 + 6.75 = 13.50

13.50 + 9.50 = 23.00


Donna guessed there were 400 jellybeans in a jar. There were really 480 jellybeans. What is her prevent of error

Answers

Answer:

Step-by-step explanation:

Donna guessed 400 but there were 480 so she was off by 80


Answer:
Need help with this plz

Answers

Answer:

(a) 3x+28+5x+52+2x-10=180 (∠ sum of Δ)

                          10x+70=180

                                 10x=110

                                     x=11

(b) ∠C = 2x-10

           = 2(11)-10

           =  12°

Alison rolls two number cubes 90 times. How many times is he going to get a sum of 3?

Answers

15

Step-by-step explanation:

Answer:

I think 30 im so sure

Step-by-step explanation:

uhm i think its 90 divided by 3 cuz i looked it up and got 30 so yea bye


Write the number 16408 in words.

Answers

Answer:

Step-by-step explanation:

Sixteen thousand, four hundred and eight

Hope this helps!

The histogram on the left shows the number of hours students in a British Literature class read last week, rounded to the nearest hour. Which of the following statements offers the best description of the median number of hours students in the class read last week?​

Answers

Answer:The median number of hours is between 5 and 9

Step-by-step explanation:

The median number of hours students in the class read last week is between 5 and 9

Histogram A histogram is an approximate representation of the distribution of numerical dataHistogram is a diagram consisting of rectangles whose area is proportional to the frequency of a variable and whose width is equal to the class interval.

How to solve this problem?

The steps are as follow:

Give class interval; Hours of reading

0-4;20

5-9;24  32th median

10-14;10

15-19;4

20-24;2

25-29;3    ∑ = 63 i.e. odd

M = 1/2 (63+1) = 32th observation

So the median number of hours students in the class read last week is between 5 and 9

Learn more about Histogram here:

https://brainly.com/question/2962546

#SPJ2

guys help. show work please

Answers

9514 1404 393

Answer:

  2^17×3^7

Step-by-step explanation:

The applicable rules of exponents are ...

  (a^b)^c = a^(bc)

  (a^b)(a^c) = a^(b+c)

__

  [tex](2^9\times3^5)\times(2^4\times3)^2=2^9\times3^5\times2^{4\cdot2}\times3^2\\\\=2^{9+8}\times3^{5+2}=\boxed{2^{17}\times3^7}[/tex]

A recent drug survey showed an increase in use of drugs and alcohol among local high school students as compared to the national percent. Suppose that a survey of 100 local youths and 100 national youths is conducted to see if the percentage of drug and alcohol use is higher locally than nationally. Locally, 65 seniors reported using drugs or alcohol within the past month, while 61 national seniors reported using them. Conduct a hypothesis test at the 5% level.

Answers

Answer:

The answer is "0.586"

Step-by-step explanation:

Please find the solution to the given question:  

[tex]z= \frac{0.65-0.61}{\sqrt{\frac{0.65 \times 0.35}{100}+\frac{0.61 \times 0.39}{100}}}[/tex]

  [tex]= \frac{0.04}{\sqrt{\frac{0.2275}{100}+\frac{0.2379}{100}}}\\\\= \frac{0.04}{\sqrt{0.002275+0.002379}}\\\\= \frac{0.04}{\sqrt{0.004654}}\\\\= \frac{0.04}{0.0682202316}\\\\=0.586336327\\\\[/tex]

Amber has determined that the experimental probability of making a free throw in basketball is 12/15. What it the probability of missing a basket?

Answers

Answer:

3/15, 1/5, or 0.2 probability. Highly unlikely

Explanation:

12/15 is the shown probability. The leftover is 3/15, so 3/15 is the chance of missing the basket.

~Hope this helps~

Jamie has white, blue, and pink shirts. She has black, white, and beige pants. She has black and brown shoes. If she chooses one shirt,

one pair of pants, and one pair of shoes, how many unique outfits can she make?

Answers

Answer:

She can make [tex]18[/tex] unique outfits.

Step-by-step explanation:

Jamie has [tex]3[/tex] options to choose from for a shirt (white, blue, or pink), [tex]3[/tex] options to choose from for pants (black, white, or beige), and [tex]2[/tex] options to choose from for shoes (black or brown). Therefore, she can make [tex]3*3*2=18[/tex] unique outfits from these options. Hope this helps!

J.B. earns $16.60 an hour with time-and-a-half for hours worked over 8 a day. His

hours for a week are 8.25, 8.25, 9.5, 11.5, and 7.25. Determine his gross earnings

for that week.

Answers

Answer:

The gross pay for the week is $701.32

Step-by-step explanation:

J.B. earns $16.60 per hour, and half of it ($16.60/2) for each hour worked over the 8th-hour range.

On the first day, he works 8.25 hours.

We have 8 hours, plus 0.25 hours past the limit, then for the first day he wins:

8*$16.60 + 0.25*($16.60/2) = $134.86

On the second day, he works again 8.25 hours, so he wins the same amount $134.86

On the third day, he works 9.5 hours, then:

We have 8 hours plus 1.5 hours past the limit, then for this day he wins:

8*$16.60 + 1.5*$16.60/2 = $145.25

On the fourth day, he works 11.5 hours, then:

We have 8 hours plus 3.5 hours pást the limit, then for this day he wins:

8*$16.60 + 3.5*$16.60/2 = $161.85

For the last day, he works 7.25 hours, (in this case, we do not have hours worked over the 8-hour limit) then this day he wins:

7.25*$16.60 = $124.50

The gross pay for this week will be equal to:

G = $134.86 + $134.86 +  $145.25 + $161.85 + $124.50 = $701.32

The area of a circle is 100π ft². What is the circumference, in feet? Express your answer in terms of π.

Answers

Answer:

Answer: 20pi is the circumference

Step-by-step explanation:

The circumference of the circle is 20π ft when the area of the circle is 100π ft².

What is circle?

A circle is a locus of the points drawn at an equidistant from a fixed point. The fixed point is called Centre of the circle and the distance from the fixed point to the circle is called radius.

Given, area of a circle is 100π.

Therefore,  πr² =100π, where r is the radius of the circle.

So, r²=100

Thus, r= 10 ft.

now, circumference = 2πr

                                  =2π×10

                                  =20π ft.

To learn more about circle click here:

https://brainly.com/question/21280267

#SPJ3

In the slope 5/9, the 5 indicates to

A. Rise 5 units
B. Run 5 units
C. Fall 5 units

Answers

A Bc it rise 5 units

Help plz:)))I’ll mark u Brainliest

Answers

Answer:

18 foot ladder will reach at 14.4 feet up the wall.

Step-by-step explanation:

Triangles formed by the ladder against wall and the ground are similar,

ΔABC ~ ΔDEC

Therefore, their corresponding sides will be proportional.

[tex]\frac{AB}{DE}= \frac{BC}{EC}= \frac{AC}{DC}[/tex]

[tex]\frac{AB}{DE}= \frac{BC}{EC}[/tex]

[tex]\frac{18}{10}=\frac{BC}{8}[/tex]

BC = [tex]\frac{18\times 8}{10}[/tex]

     = 14.4

Therefore, 18 foot ladder will reach at 14.4 feet up the wall.

write the equation for the function of the graph below​

Answers

Answer:  (1) [tex]y= \sqrt{x+2}-2[/tex]   (2) y = (x - 3)³

                    D: x ≥ -2  [-2, ∞)      D: -∞ < x < ∞ (-∞, ∞)

                    R: y ≥ -2  [-2, ∞)      R: -∞ < y < ∞ (-∞, ∞)

                    Inc: x > 2 (2, ∞)      Inc: (-∞, 3) ∪ (3, ∞)

                    Dec: never             Dec: never

Step-by-step explanation:

NOTES:

Domain is the x-values.

Range is the y-values.

From left to right: increasing when going up and decreasing when going down. The anchor and turning point are not considered to be increasing or decreasing.

-1/2x-(1/2x+4)+12=17x-6(3x+5/6) What value of x makes the equation true? Show your work please.

Answers

Answer:

No Solution

Step-by-step explanation:

Keep in mind that if the grouping isn't right, then my result will be wrong:

-1/2x-(1/2x+4)+12=17x-6(3x+5/6)

Let me just change the formatting for convenience, and then begin the calculations:

[tex]-\frac{1}{2}x-\left(\frac{1}{2}x+4\right)+12=17x-6\left(3x+\frac{5}{6}\right)\:[/tex]

[tex]-\frac{1}{2}x-\frac{1}{2}x-4+12=17x-18x-5[/tex]

[tex]-2\cdot \frac{1}{2}x-4+12=17x-18x-5[/tex]

[tex]-x-4+12=17x-18x-5[/tex]

[tex]-x+8=17x-18x-5[/tex]

[tex]-x+8=-x-5[/tex]

[tex]-x=-x-13[/tex]

[tex]0=-13[/tex]

Result: No Solution

Find a cubic polynomial that goes through points (4, – 22) and (3, - 26) and has tangents with slopes
respectively 11 and — 2 there. Check your work with a graphing utility.
f() =

Answers

Let f(x) = ax ³ + bx ² + cx + d.

The graph of f(x) passes through (4, -22) and (3, -26), which means f (4) = -22 and f (3) = -26, so that

64a + 16b + 4c + d = -22

27a + 9b + 3c + d = -26

When the question says it has tangents at some point, I take that to mean the slope of the tangent line at that point is the given number. So f ' (4) = 11 and f ' (3) = -2. We have

f '(x) = 3ax ²+ 2bx + c

so that

48a + 8b + c = 11

27a + 6b + c = -2

Solve the system:

• Eliminate d :

(64a + 16b + 4c + d) - (27a + 9b + 3c + d) = -22 - (-26)

→   37a + 7b + c = 4

• Eliminate c :

(48a + 8b + c) - (27a + 6b + c) = 11 - (-2)

→   21a + 2b = 13

(48a + 8b + c) - (37a + 7b + c) = 11 - 4

→   11a + b = 7

• Eliminate b, then solve for a and the other variables:

(21a + 2b) - 2 (11a + b) = 13 - 2 (7)

-a = -1

a = 1   →   b = -4   →   c = -5   →   d = -2

Then

f(x) = x ³ - 4x ² - 5x - 2

WILL GIVE BRAINLIEST

Answers

Answer:

[tex]\frac{y}{8}[/tex]

Step-by-step explanation:

quotient means division

the / means divide

Answer:

y/8

Step-by-step explanation:

quotient means divide, so y divided by 8, or y/8

Exercise 2.4.2: Proving statements about rational numbers with direct proofs. About Prove each of the following statements using a direct proof. (a) The product of two rational numbers is a rational number. Solution (b) The quotient of a rational number and a non-zero rational number is a rational number. Solution (c) If x and y are rational numbers then is also a rational number.

Answers

Answer:

See Explanation

Step-by-step explanation:

(a) Proof: Product of two rational numbers

Using direct proofs.

Let the two rational numbers be A and B.

Such that:

[tex]A = \frac{1}{2}[/tex]

[tex]B = \frac{2}{3}[/tex]

The product:

[tex]A * B = \frac{1}{2} * \frac{2}{3}[/tex]

[tex]A * B = \frac{1}{1} * \frac{1}{3}[/tex]

[tex]A * B = 1 * \frac{1}{3}[/tex]

[tex]A * B = \frac{1}{3}[/tex]

Proved, because 1/3 is rational

(b) Proof: Quotient of a rational number and a non-zero rational number

Using direct proofs.

Let the two rational numbers be A and B.

Such that:

[tex]A = \frac{1}{2}[/tex]

[tex]B = \frac{2}{3}[/tex]

The quotient:

[tex]A / B = \frac{1}{2} / \frac{2}{3}[/tex]

Express as product

[tex]A / B = \frac{1}{2} / \frac{3}{2}[/tex]

[tex]A / B = \frac{1*3}{2*2}[/tex]

[tex]A / B = \frac{3}{4}[/tex]

Proved, because 3/4 is rational

(c) x + y is rational (missing from the question)

Using direct proofs.

Let x and y be

Such that:

[tex]x = \frac{1}{2}[/tex]

[tex]y = \frac{2}{3}[/tex]

The sum:

[tex]x + y = \frac{1}{2} + \frac{2}{3}[/tex]

Take LCM

[tex]x + y = \frac{3+4}{6}[/tex]

[tex]x + y = \frac{7}{6}[/tex]

Proved, because 7/6 is rational

The above proof works for all values of A, B, x and y; as long as they are rational values

PLZ HELP ME YOU GUYS ARE MY ONLY HOPE!!! I WILL GIVE BRAINLIEST PLZ ANSWER<3
2a+3c=5a+2c=1

There is an ordered pair that is the solution of the system of equations shown. The value of one of the variables in the solution of the system can be obtained by using which of the following substitutions?

Answers

Answer:

a = 9, c = -4?

Step-by-step explanation:

i think these are the two equations, let me know if i'm wrong

2a + 3c = 5; a + 2c = 1

rearrange second equation : a = -2c +1

plug in:

2(-2c +1) +3c = 5

-4c +1 +3c = 5

-c +1 = 5

-c = 4

c = -4

a + 2(-4) = 1

a - 8 = 1

a = 9

a=9, c=4 is the answer good luck

A company has a linear total cost function and has determined that over the next three
months it can produce 13,000 units at a total cost of $226,000. This same manufacturer
can produce 16,000 units at a total cost of $296,000. The sales price per unit is $31.25.
i. Find the revenue, cost, and profit functions using q for number of units.
ii. Find the marginal cost, average cost and fixed cost.
iii. Find break-even quantity.

Answers

I am so sorry but I don’t really know the answrr

Zack has an old car. He wants to sell it for 60% off the current price. The market price is $500. How much money would he receive in exchange for the car if he were able to sell it at that rate? What if he wanted to sell it for 25% more than the market price?

Answers

300 and 625

60% of 500 is 300

So if Zack wanted to sell the car 60% of the markets price, it would be 300.

125% of 500 is 625

If Zack wanted to sell the car for 25 more than the price at market, it would be 125%, and 125% of 500 is 625.

Expand the expression -3 (4s+-7). Then select the correct answer.
A. 12s - 21.
B 12s + 21.
C -12s + 21.
D -12s - 21​

Answers

Answer:

I think A is the correct answer because -3+4 is 1 because 4 has greater sign + and +- gives result - thus,-3-7 gives - 21. I hope that it would be meaning ful for you ...

Find the slope of the line passing through the points (-2,-3) and (2,5) 

Answers

Answer:

Undefined

Step-by-step explanation:

Find the value of x 90, 68 2x

Answers

Answer:

2x + x +90= 180 We will add 2x + x=3x We get 3x + 90 =180 Now we subtract 3x = 180–90 ... x=30, for confirmation put value of x =30in equation and verify. LHS=RHS. Thats it. 68 views.

Given f(x)=x^3-2, find the equation of the secant line passing through (-4,f(-4)) and (2,f(2))

Answers

Given:

The function is

[tex]f(x)=x^3-2[/tex]

The secant line passing through (-4,f(-4)) and (2,f(2)).

To find:

The equation of the secant line.

Solution:

We have,

[tex]f(x)=x^3-2[/tex]

At x=-4,

[tex]f(-4)=(-4)^3-2[/tex]

[tex]f(-4)=-64-2[/tex]

[tex]f(-4)=-66[/tex]

At x=2,

[tex]f(2)=(2)^3-2[/tex]

[tex]f(2)=8-2[/tex]

[tex]f(2)=6[/tex]

The secant line passes through the points (-4,-66) and (2,6). So, the equation of the secant line is

[tex]y-y_1=\dfrac{y_2-y_1}{x_2-x_1}(x-x_1)[/tex]

[tex]y-(-66)=\dfrac{6-(-66)}{2-(-4)}(x-(-4))[/tex]

[tex]y+66=\dfrac{6+66}{2+4}(x+4)[/tex]

[tex]y+66=\dfrac{72}{6}(x+4)[/tex]

On further simplification, we get

[tex]y+66=12(x+4)[/tex]

[tex]y+66=12x+48[/tex]

[tex]y=12x+48-66[/tex]

[tex]y=12x-18[/tex]

Therefor, the equation of the secant line is [tex]y=12x-18[/tex].

Jenny sold 17 less than 3 times the number of quilts Laura sold. Jenny sold
4 quilts. Select whether each statement is true or false. ZEE.4. ZEE 4a

Answers

Answer:

the statement would be false.

Step-by-step explanation:

Jenny sold 17 less than 3 times lauras 4 quilts. So that would make Laura have 12 and 12<17.

Other Questions
spanish 1 ; i need help with adjectives Order: Gentamicin (garamycin)55 mg IM q8hr.Available: Gentamicin(garamycin) 80 mg per 2 ml.How many ml will the nurseadminister for one dose?How many ml will the nurseadminister for the day? The diagram shows part of a frogs body. The structure labeled X most likely helps with which activity?swimmingclimbingjumpingfertilizing David measures a line to be 4.87 in long. If the actual measurement is 5 in, find David's relative error to the nearest hundredth.Ill give brainilest to whoever helps what is 3/4/5-2/3/5... Is this possible?a=ba=aba-b=ab-b(a+b)(a-b)=b(a-b)(a+b)=ba+a=a2a=a2=1 What is an equation of the line that passes through the points (1,6) and (1, 2)? A chef has 5 blocks of butter. Each block weighs 1 pound. She cuts each block into 1/3How many 1/3 pound pieces of butter does the chef have?The chef has __________one-third-pound pieces of butter. What is the velocity of a 750 kg object at a height of 0m if its ME is 3590J? what is the simplifed form of the fraction below?24/30 A woman can bicycle 80 miles in the same time as it takes her to walk 16 miles. She can ride 8 mph faster than she can walk. How fast can she walk Enter the ordered pair that is the solution to the system of equations graphed below.y = 1/3x +4y = 2x - 1 what is the equation of the line that passes through the point (-5,1) and has a slope of -1/5? A right triangle has one leg that is 4 more than twice the other. The area of the triangle is 35. Find the length of the sides. Can I get some help please, hurry!!???!!?? What does dynamic stretches mean? study the Banking Sector of india and prepare a report on it as per your understanding HELP ME PLEASE HOW DO ALL OF THE PARTS OF THE CELL HELP TOMAKE IT FUNCTION AS A WHOLE?(5 sentences minimum with 3 pieces of evidence and reasoning) Round 47,637 to the nearest hundred. why is poop good for other mammals